Prove that two sequences converge to the same limit.












0














I encountered this question in my homework:



$$a_1=x, b_1=y, \
a_{n+1} =frac{a_n+b_n}{2}, b_{n+1}= sqrt{a_nb_n}, nin mathbb{N}$$



Given $x,y$ positive constants.
I have to prove that they both converge to the same limit $L$.



I know that in order to prove that a sequence converges, it has to be (for example) Monotonically increasing and that $|a_n|<K$



I'm having difficulties proving with induction that either of the sequences is monotonic... how do I do so when the first sequence depends on the other? I know that by using the average inequality I get that $a_{n+1} > b_{n+1}$. How do I continue from here?



Thank you for your time and help!










share|cite|improve this question
























  • $(sqrt{a_n}+sqrt{b_n})^2=a_n+b_n+2sqrt{a_nb_n} geq 0$ From here we get that $frac{a_n+b_n}{2}+sqrt{a_nb_n} geq 0$
    – mathnoob
    Nov 23 '18 at 14:00
















0














I encountered this question in my homework:



$$a_1=x, b_1=y, \
a_{n+1} =frac{a_n+b_n}{2}, b_{n+1}= sqrt{a_nb_n}, nin mathbb{N}$$



Given $x,y$ positive constants.
I have to prove that they both converge to the same limit $L$.



I know that in order to prove that a sequence converges, it has to be (for example) Monotonically increasing and that $|a_n|<K$



I'm having difficulties proving with induction that either of the sequences is monotonic... how do I do so when the first sequence depends on the other? I know that by using the average inequality I get that $a_{n+1} > b_{n+1}$. How do I continue from here?



Thank you for your time and help!










share|cite|improve this question
























  • $(sqrt{a_n}+sqrt{b_n})^2=a_n+b_n+2sqrt{a_nb_n} geq 0$ From here we get that $frac{a_n+b_n}{2}+sqrt{a_nb_n} geq 0$
    – mathnoob
    Nov 23 '18 at 14:00














0












0








0







I encountered this question in my homework:



$$a_1=x, b_1=y, \
a_{n+1} =frac{a_n+b_n}{2}, b_{n+1}= sqrt{a_nb_n}, nin mathbb{N}$$



Given $x,y$ positive constants.
I have to prove that they both converge to the same limit $L$.



I know that in order to prove that a sequence converges, it has to be (for example) Monotonically increasing and that $|a_n|<K$



I'm having difficulties proving with induction that either of the sequences is monotonic... how do I do so when the first sequence depends on the other? I know that by using the average inequality I get that $a_{n+1} > b_{n+1}$. How do I continue from here?



Thank you for your time and help!










share|cite|improve this question















I encountered this question in my homework:



$$a_1=x, b_1=y, \
a_{n+1} =frac{a_n+b_n}{2}, b_{n+1}= sqrt{a_nb_n}, nin mathbb{N}$$



Given $x,y$ positive constants.
I have to prove that they both converge to the same limit $L$.



I know that in order to prove that a sequence converges, it has to be (for example) Monotonically increasing and that $|a_n|<K$



I'm having difficulties proving with induction that either of the sequences is monotonic... how do I do so when the first sequence depends on the other? I know that by using the average inequality I get that $a_{n+1} > b_{n+1}$. How do I continue from here?



Thank you for your time and help!







sequences-and-series convergence induction






share|cite|improve this question















share|cite|improve this question













share|cite|improve this question




share|cite|improve this question








edited Nov 23 '18 at 13:57









Mason

1,9591530




1,9591530










asked Nov 23 '18 at 13:40









Buk LauBuk Lau

1878




1878












  • $(sqrt{a_n}+sqrt{b_n})^2=a_n+b_n+2sqrt{a_nb_n} geq 0$ From here we get that $frac{a_n+b_n}{2}+sqrt{a_nb_n} geq 0$
    – mathnoob
    Nov 23 '18 at 14:00


















  • $(sqrt{a_n}+sqrt{b_n})^2=a_n+b_n+2sqrt{a_nb_n} geq 0$ From here we get that $frac{a_n+b_n}{2}+sqrt{a_nb_n} geq 0$
    – mathnoob
    Nov 23 '18 at 14:00
















$(sqrt{a_n}+sqrt{b_n})^2=a_n+b_n+2sqrt{a_nb_n} geq 0$ From here we get that $frac{a_n+b_n}{2}+sqrt{a_nb_n} geq 0$
– mathnoob
Nov 23 '18 at 14:00




$(sqrt{a_n}+sqrt{b_n})^2=a_n+b_n+2sqrt{a_nb_n} geq 0$ From here we get that $frac{a_n+b_n}{2}+sqrt{a_nb_n} geq 0$
– mathnoob
Nov 23 '18 at 14:00










3 Answers
3






active

oldest

votes


















1














By AM-GM inequality,



$$b_{n+1} le a_{n+1}$$



Also, if $ b_n le a_n$, then $a_{n+1} le a_n$ since $a_{n+1}$ is the arithmetic means between $a_n$ and $b_n$.



Also if $b_n le a_n$, then $b_{n+1} ge b_n$ since $b_{n+1}$ is the geometric means between $a_n$ and $b_n$.



Hence we have $$b_{n+1} le b_{n+2} le a_{n+2} le a_{n+1}$$



Both sequence converges.
From
$$ a=frac{a+b}2$$



we can deduce that $a=b$.






share|cite|improve this answer





















  • Does that hold for any n? Or do I have to do it through induction? Do I have to assume that x>y or the other way around? Thank you!
    – Buk Lau
    Nov 23 '18 at 14:16










  • try to prove by induction from $n ge 2$ onwards. we do not need assumption of $x>y$. it always hold for any positive pairs.
    – Siong Thye Goh
    Nov 23 '18 at 14:18










  • I never encountered such a sequence that relies on another sequencem I don't know how to even start, like how do I do it just for an for example? how do I take bn out of the picture?
    – Buk Lau
    Nov 23 '18 at 14:21










  • perhaps go slow, first check that $a_n$ and $b_n$ are positive. After that check that $a_{n+1} < b_{n+1}$. after that then prove the monotonicity. ping if you are stuck and update me which step are u stuck at?
    – Siong Thye Goh
    Nov 23 '18 at 14:28










  • imgur.com/a/TaaijH9 is this the way to do it? Or did I write nonsense? Sorry if so!! Thank you.
    – Buk Lau
    Nov 23 '18 at 15:01



















1














if $x>y$, first observe that $x ge a_nge a_{n+1} ge b_{n+1} ge b_n ge y$ for all $ninmathbb{N}$



So both ${a_n}$ and ${b_n}$ converge. (as they are monotonic bounded)



Also $|a_n-b_n|le|frac{a_{n-1}+b_{n-1}}{2}-b_{n-1}|le frac{1}{2}|a_{n-1}-b_{n-1}|le frac{1}{2^{n-1}} |a_1-b_1| to 0 $ as $nto infty$



So ${a_n}$ and ${b_n}$ converge at same limit.



if $y>x$ then $y ge a_nge a_{n+1} ge b_{n+1} ge b_n ge x$ for all $n>1$.






share|cite|improve this answer





















  • I did get to that inequity chain but I couldn't tell if it holds for all n, I just assumed it did, since all I could say for sure is that a2>a1 (when x>y). I know that it's true but do I have to show it for all n or can I just say it the way you did? Thank you!
    – Buk Lau
    Nov 23 '18 at 14:40










  • As $x,y$ are both positive it is the standard AM-GM inequality. So you also can use the result in formal proof.
    – Offlaw
    Nov 23 '18 at 15:04










  • This is exactly what I did but I thought that I still have to prove that it does hold for all n, I then got that they converge to the same limit the same way Siong Thye Goh mentioned in his answer. will this be enough as a formal proof? Thank you for your time!
    – Buk Lau
    Nov 23 '18 at 15:16



















0














$|a_{n+1}-b_{n+1}|=left|frac{a_n+b_n}{2}-sqrt{a_nb_n}right|=1/2left|a_n+b_n-2sqrt{a_nb_n}right|=1/2(sqrt{a_n}-sqrt{b_n})^2=(1/2)(1/4)(sqrt{a_{n-1}}-sqrt{b_{n-1}})^4=2^{-1-2}(sqrt{a_{n-1}}-sqrt{b_{n-1}})^{2times 2}=ldots=2^{-1-2-ldots-n}(sqrt{x}-sqrt{y})^{2n}=2^{-n(n+1)/2}(sqrt{x}-sqrt{y})^{2n}rightarrow 0$ as $nrightarrowinfty.$






share|cite|improve this answer























  • How did you get the third equality?
    – mathnoob
    Nov 23 '18 at 14:09











Your Answer





StackExchange.ifUsing("editor", function () {
return StackExchange.using("mathjaxEditing", function () {
StackExchange.MarkdownEditor.creationCallbacks.add(function (editor, postfix) {
StackExchange.mathjaxEditing.prepareWmdForMathJax(editor, postfix, [["$", "$"], ["\\(","\\)"]]);
});
});
}, "mathjax-editing");

StackExchange.ready(function() {
var channelOptions = {
tags: "".split(" "),
id: "69"
};
initTagRenderer("".split(" "), "".split(" "), channelOptions);

StackExchange.using("externalEditor", function() {
// Have to fire editor after snippets, if snippets enabled
if (StackExchange.settings.snippets.snippetsEnabled) {
StackExchange.using("snippets", function() {
createEditor();
});
}
else {
createEditor();
}
});

function createEditor() {
StackExchange.prepareEditor({
heartbeatType: 'answer',
autoActivateHeartbeat: false,
convertImagesToLinks: true,
noModals: true,
showLowRepImageUploadWarning: true,
reputationToPostImages: 10,
bindNavPrevention: true,
postfix: "",
imageUploader: {
brandingHtml: "Powered by u003ca class="icon-imgur-white" href="https://imgur.com/"u003eu003c/au003e",
contentPolicyHtml: "User contributions licensed under u003ca href="https://creativecommons.org/licenses/by-sa/3.0/"u003ecc by-sa 3.0 with attribution requiredu003c/au003e u003ca href="https://stackoverflow.com/legal/content-policy"u003e(content policy)u003c/au003e",
allowUrls: true
},
noCode: true, onDemand: true,
discardSelector: ".discard-answer"
,immediatelyShowMarkdownHelp:true
});


}
});














draft saved

draft discarded


















StackExchange.ready(
function () {
StackExchange.openid.initPostLogin('.new-post-login', 'https%3a%2f%2fmath.stackexchange.com%2fquestions%2f3010377%2fprove-that-two-sequences-converge-to-the-same-limit%23new-answer', 'question_page');
}
);

Post as a guest















Required, but never shown

























3 Answers
3






active

oldest

votes








3 Answers
3






active

oldest

votes









active

oldest

votes






active

oldest

votes









1














By AM-GM inequality,



$$b_{n+1} le a_{n+1}$$



Also, if $ b_n le a_n$, then $a_{n+1} le a_n$ since $a_{n+1}$ is the arithmetic means between $a_n$ and $b_n$.



Also if $b_n le a_n$, then $b_{n+1} ge b_n$ since $b_{n+1}$ is the geometric means between $a_n$ and $b_n$.



Hence we have $$b_{n+1} le b_{n+2} le a_{n+2} le a_{n+1}$$



Both sequence converges.
From
$$ a=frac{a+b}2$$



we can deduce that $a=b$.






share|cite|improve this answer





















  • Does that hold for any n? Or do I have to do it through induction? Do I have to assume that x>y or the other way around? Thank you!
    – Buk Lau
    Nov 23 '18 at 14:16










  • try to prove by induction from $n ge 2$ onwards. we do not need assumption of $x>y$. it always hold for any positive pairs.
    – Siong Thye Goh
    Nov 23 '18 at 14:18










  • I never encountered such a sequence that relies on another sequencem I don't know how to even start, like how do I do it just for an for example? how do I take bn out of the picture?
    – Buk Lau
    Nov 23 '18 at 14:21










  • perhaps go slow, first check that $a_n$ and $b_n$ are positive. After that check that $a_{n+1} < b_{n+1}$. after that then prove the monotonicity. ping if you are stuck and update me which step are u stuck at?
    – Siong Thye Goh
    Nov 23 '18 at 14:28










  • imgur.com/a/TaaijH9 is this the way to do it? Or did I write nonsense? Sorry if so!! Thank you.
    – Buk Lau
    Nov 23 '18 at 15:01
















1














By AM-GM inequality,



$$b_{n+1} le a_{n+1}$$



Also, if $ b_n le a_n$, then $a_{n+1} le a_n$ since $a_{n+1}$ is the arithmetic means between $a_n$ and $b_n$.



Also if $b_n le a_n$, then $b_{n+1} ge b_n$ since $b_{n+1}$ is the geometric means between $a_n$ and $b_n$.



Hence we have $$b_{n+1} le b_{n+2} le a_{n+2} le a_{n+1}$$



Both sequence converges.
From
$$ a=frac{a+b}2$$



we can deduce that $a=b$.






share|cite|improve this answer





















  • Does that hold for any n? Or do I have to do it through induction? Do I have to assume that x>y or the other way around? Thank you!
    – Buk Lau
    Nov 23 '18 at 14:16










  • try to prove by induction from $n ge 2$ onwards. we do not need assumption of $x>y$. it always hold for any positive pairs.
    – Siong Thye Goh
    Nov 23 '18 at 14:18










  • I never encountered such a sequence that relies on another sequencem I don't know how to even start, like how do I do it just for an for example? how do I take bn out of the picture?
    – Buk Lau
    Nov 23 '18 at 14:21










  • perhaps go slow, first check that $a_n$ and $b_n$ are positive. After that check that $a_{n+1} < b_{n+1}$. after that then prove the monotonicity. ping if you are stuck and update me which step are u stuck at?
    – Siong Thye Goh
    Nov 23 '18 at 14:28










  • imgur.com/a/TaaijH9 is this the way to do it? Or did I write nonsense? Sorry if so!! Thank you.
    – Buk Lau
    Nov 23 '18 at 15:01














1












1








1






By AM-GM inequality,



$$b_{n+1} le a_{n+1}$$



Also, if $ b_n le a_n$, then $a_{n+1} le a_n$ since $a_{n+1}$ is the arithmetic means between $a_n$ and $b_n$.



Also if $b_n le a_n$, then $b_{n+1} ge b_n$ since $b_{n+1}$ is the geometric means between $a_n$ and $b_n$.



Hence we have $$b_{n+1} le b_{n+2} le a_{n+2} le a_{n+1}$$



Both sequence converges.
From
$$ a=frac{a+b}2$$



we can deduce that $a=b$.






share|cite|improve this answer












By AM-GM inequality,



$$b_{n+1} le a_{n+1}$$



Also, if $ b_n le a_n$, then $a_{n+1} le a_n$ since $a_{n+1}$ is the arithmetic means between $a_n$ and $b_n$.



Also if $b_n le a_n$, then $b_{n+1} ge b_n$ since $b_{n+1}$ is the geometric means between $a_n$ and $b_n$.



Hence we have $$b_{n+1} le b_{n+2} le a_{n+2} le a_{n+1}$$



Both sequence converges.
From
$$ a=frac{a+b}2$$



we can deduce that $a=b$.







share|cite|improve this answer












share|cite|improve this answer



share|cite|improve this answer










answered Nov 23 '18 at 14:12









Siong Thye GohSiong Thye Goh

100k1465117




100k1465117












  • Does that hold for any n? Or do I have to do it through induction? Do I have to assume that x>y or the other way around? Thank you!
    – Buk Lau
    Nov 23 '18 at 14:16










  • try to prove by induction from $n ge 2$ onwards. we do not need assumption of $x>y$. it always hold for any positive pairs.
    – Siong Thye Goh
    Nov 23 '18 at 14:18










  • I never encountered such a sequence that relies on another sequencem I don't know how to even start, like how do I do it just for an for example? how do I take bn out of the picture?
    – Buk Lau
    Nov 23 '18 at 14:21










  • perhaps go slow, first check that $a_n$ and $b_n$ are positive. After that check that $a_{n+1} < b_{n+1}$. after that then prove the monotonicity. ping if you are stuck and update me which step are u stuck at?
    – Siong Thye Goh
    Nov 23 '18 at 14:28










  • imgur.com/a/TaaijH9 is this the way to do it? Or did I write nonsense? Sorry if so!! Thank you.
    – Buk Lau
    Nov 23 '18 at 15:01


















  • Does that hold for any n? Or do I have to do it through induction? Do I have to assume that x>y or the other way around? Thank you!
    – Buk Lau
    Nov 23 '18 at 14:16










  • try to prove by induction from $n ge 2$ onwards. we do not need assumption of $x>y$. it always hold for any positive pairs.
    – Siong Thye Goh
    Nov 23 '18 at 14:18










  • I never encountered such a sequence that relies on another sequencem I don't know how to even start, like how do I do it just for an for example? how do I take bn out of the picture?
    – Buk Lau
    Nov 23 '18 at 14:21










  • perhaps go slow, first check that $a_n$ and $b_n$ are positive. After that check that $a_{n+1} < b_{n+1}$. after that then prove the monotonicity. ping if you are stuck and update me which step are u stuck at?
    – Siong Thye Goh
    Nov 23 '18 at 14:28










  • imgur.com/a/TaaijH9 is this the way to do it? Or did I write nonsense? Sorry if so!! Thank you.
    – Buk Lau
    Nov 23 '18 at 15:01
















Does that hold for any n? Or do I have to do it through induction? Do I have to assume that x>y or the other way around? Thank you!
– Buk Lau
Nov 23 '18 at 14:16




Does that hold for any n? Or do I have to do it through induction? Do I have to assume that x>y or the other way around? Thank you!
– Buk Lau
Nov 23 '18 at 14:16












try to prove by induction from $n ge 2$ onwards. we do not need assumption of $x>y$. it always hold for any positive pairs.
– Siong Thye Goh
Nov 23 '18 at 14:18




try to prove by induction from $n ge 2$ onwards. we do not need assumption of $x>y$. it always hold for any positive pairs.
– Siong Thye Goh
Nov 23 '18 at 14:18












I never encountered such a sequence that relies on another sequencem I don't know how to even start, like how do I do it just for an for example? how do I take bn out of the picture?
– Buk Lau
Nov 23 '18 at 14:21




I never encountered such a sequence that relies on another sequencem I don't know how to even start, like how do I do it just for an for example? how do I take bn out of the picture?
– Buk Lau
Nov 23 '18 at 14:21












perhaps go slow, first check that $a_n$ and $b_n$ are positive. After that check that $a_{n+1} < b_{n+1}$. after that then prove the monotonicity. ping if you are stuck and update me which step are u stuck at?
– Siong Thye Goh
Nov 23 '18 at 14:28




perhaps go slow, first check that $a_n$ and $b_n$ are positive. After that check that $a_{n+1} < b_{n+1}$. after that then prove the monotonicity. ping if you are stuck and update me which step are u stuck at?
– Siong Thye Goh
Nov 23 '18 at 14:28












imgur.com/a/TaaijH9 is this the way to do it? Or did I write nonsense? Sorry if so!! Thank you.
– Buk Lau
Nov 23 '18 at 15:01




imgur.com/a/TaaijH9 is this the way to do it? Or did I write nonsense? Sorry if so!! Thank you.
– Buk Lau
Nov 23 '18 at 15:01











1














if $x>y$, first observe that $x ge a_nge a_{n+1} ge b_{n+1} ge b_n ge y$ for all $ninmathbb{N}$



So both ${a_n}$ and ${b_n}$ converge. (as they are monotonic bounded)



Also $|a_n-b_n|le|frac{a_{n-1}+b_{n-1}}{2}-b_{n-1}|le frac{1}{2}|a_{n-1}-b_{n-1}|le frac{1}{2^{n-1}} |a_1-b_1| to 0 $ as $nto infty$



So ${a_n}$ and ${b_n}$ converge at same limit.



if $y>x$ then $y ge a_nge a_{n+1} ge b_{n+1} ge b_n ge x$ for all $n>1$.






share|cite|improve this answer





















  • I did get to that inequity chain but I couldn't tell if it holds for all n, I just assumed it did, since all I could say for sure is that a2>a1 (when x>y). I know that it's true but do I have to show it for all n or can I just say it the way you did? Thank you!
    – Buk Lau
    Nov 23 '18 at 14:40










  • As $x,y$ are both positive it is the standard AM-GM inequality. So you also can use the result in formal proof.
    – Offlaw
    Nov 23 '18 at 15:04










  • This is exactly what I did but I thought that I still have to prove that it does hold for all n, I then got that they converge to the same limit the same way Siong Thye Goh mentioned in his answer. will this be enough as a formal proof? Thank you for your time!
    – Buk Lau
    Nov 23 '18 at 15:16
















1














if $x>y$, first observe that $x ge a_nge a_{n+1} ge b_{n+1} ge b_n ge y$ for all $ninmathbb{N}$



So both ${a_n}$ and ${b_n}$ converge. (as they are monotonic bounded)



Also $|a_n-b_n|le|frac{a_{n-1}+b_{n-1}}{2}-b_{n-1}|le frac{1}{2}|a_{n-1}-b_{n-1}|le frac{1}{2^{n-1}} |a_1-b_1| to 0 $ as $nto infty$



So ${a_n}$ and ${b_n}$ converge at same limit.



if $y>x$ then $y ge a_nge a_{n+1} ge b_{n+1} ge b_n ge x$ for all $n>1$.






share|cite|improve this answer





















  • I did get to that inequity chain but I couldn't tell if it holds for all n, I just assumed it did, since all I could say for sure is that a2>a1 (when x>y). I know that it's true but do I have to show it for all n or can I just say it the way you did? Thank you!
    – Buk Lau
    Nov 23 '18 at 14:40










  • As $x,y$ are both positive it is the standard AM-GM inequality. So you also can use the result in formal proof.
    – Offlaw
    Nov 23 '18 at 15:04










  • This is exactly what I did but I thought that I still have to prove that it does hold for all n, I then got that they converge to the same limit the same way Siong Thye Goh mentioned in his answer. will this be enough as a formal proof? Thank you for your time!
    – Buk Lau
    Nov 23 '18 at 15:16














1












1








1






if $x>y$, first observe that $x ge a_nge a_{n+1} ge b_{n+1} ge b_n ge y$ for all $ninmathbb{N}$



So both ${a_n}$ and ${b_n}$ converge. (as they are monotonic bounded)



Also $|a_n-b_n|le|frac{a_{n-1}+b_{n-1}}{2}-b_{n-1}|le frac{1}{2}|a_{n-1}-b_{n-1}|le frac{1}{2^{n-1}} |a_1-b_1| to 0 $ as $nto infty$



So ${a_n}$ and ${b_n}$ converge at same limit.



if $y>x$ then $y ge a_nge a_{n+1} ge b_{n+1} ge b_n ge x$ for all $n>1$.






share|cite|improve this answer












if $x>y$, first observe that $x ge a_nge a_{n+1} ge b_{n+1} ge b_n ge y$ for all $ninmathbb{N}$



So both ${a_n}$ and ${b_n}$ converge. (as they are monotonic bounded)



Also $|a_n-b_n|le|frac{a_{n-1}+b_{n-1}}{2}-b_{n-1}|le frac{1}{2}|a_{n-1}-b_{n-1}|le frac{1}{2^{n-1}} |a_1-b_1| to 0 $ as $nto infty$



So ${a_n}$ and ${b_n}$ converge at same limit.



if $y>x$ then $y ge a_nge a_{n+1} ge b_{n+1} ge b_n ge x$ for all $n>1$.







share|cite|improve this answer












share|cite|improve this answer



share|cite|improve this answer










answered Nov 23 '18 at 14:19









OfflawOfflaw

2649




2649












  • I did get to that inequity chain but I couldn't tell if it holds for all n, I just assumed it did, since all I could say for sure is that a2>a1 (when x>y). I know that it's true but do I have to show it for all n or can I just say it the way you did? Thank you!
    – Buk Lau
    Nov 23 '18 at 14:40










  • As $x,y$ are both positive it is the standard AM-GM inequality. So you also can use the result in formal proof.
    – Offlaw
    Nov 23 '18 at 15:04










  • This is exactly what I did but I thought that I still have to prove that it does hold for all n, I then got that they converge to the same limit the same way Siong Thye Goh mentioned in his answer. will this be enough as a formal proof? Thank you for your time!
    – Buk Lau
    Nov 23 '18 at 15:16


















  • I did get to that inequity chain but I couldn't tell if it holds for all n, I just assumed it did, since all I could say for sure is that a2>a1 (when x>y). I know that it's true but do I have to show it for all n or can I just say it the way you did? Thank you!
    – Buk Lau
    Nov 23 '18 at 14:40










  • As $x,y$ are both positive it is the standard AM-GM inequality. So you also can use the result in formal proof.
    – Offlaw
    Nov 23 '18 at 15:04










  • This is exactly what I did but I thought that I still have to prove that it does hold for all n, I then got that they converge to the same limit the same way Siong Thye Goh mentioned in his answer. will this be enough as a formal proof? Thank you for your time!
    – Buk Lau
    Nov 23 '18 at 15:16
















I did get to that inequity chain but I couldn't tell if it holds for all n, I just assumed it did, since all I could say for sure is that a2>a1 (when x>y). I know that it's true but do I have to show it for all n or can I just say it the way you did? Thank you!
– Buk Lau
Nov 23 '18 at 14:40




I did get to that inequity chain but I couldn't tell if it holds for all n, I just assumed it did, since all I could say for sure is that a2>a1 (when x>y). I know that it's true but do I have to show it for all n or can I just say it the way you did? Thank you!
– Buk Lau
Nov 23 '18 at 14:40












As $x,y$ are both positive it is the standard AM-GM inequality. So you also can use the result in formal proof.
– Offlaw
Nov 23 '18 at 15:04




As $x,y$ are both positive it is the standard AM-GM inequality. So you also can use the result in formal proof.
– Offlaw
Nov 23 '18 at 15:04












This is exactly what I did but I thought that I still have to prove that it does hold for all n, I then got that they converge to the same limit the same way Siong Thye Goh mentioned in his answer. will this be enough as a formal proof? Thank you for your time!
– Buk Lau
Nov 23 '18 at 15:16




This is exactly what I did but I thought that I still have to prove that it does hold for all n, I then got that they converge to the same limit the same way Siong Thye Goh mentioned in his answer. will this be enough as a formal proof? Thank you for your time!
– Buk Lau
Nov 23 '18 at 15:16











0














$|a_{n+1}-b_{n+1}|=left|frac{a_n+b_n}{2}-sqrt{a_nb_n}right|=1/2left|a_n+b_n-2sqrt{a_nb_n}right|=1/2(sqrt{a_n}-sqrt{b_n})^2=(1/2)(1/4)(sqrt{a_{n-1}}-sqrt{b_{n-1}})^4=2^{-1-2}(sqrt{a_{n-1}}-sqrt{b_{n-1}})^{2times 2}=ldots=2^{-1-2-ldots-n}(sqrt{x}-sqrt{y})^{2n}=2^{-n(n+1)/2}(sqrt{x}-sqrt{y})^{2n}rightarrow 0$ as $nrightarrowinfty.$






share|cite|improve this answer























  • How did you get the third equality?
    – mathnoob
    Nov 23 '18 at 14:09
















0














$|a_{n+1}-b_{n+1}|=left|frac{a_n+b_n}{2}-sqrt{a_nb_n}right|=1/2left|a_n+b_n-2sqrt{a_nb_n}right|=1/2(sqrt{a_n}-sqrt{b_n})^2=(1/2)(1/4)(sqrt{a_{n-1}}-sqrt{b_{n-1}})^4=2^{-1-2}(sqrt{a_{n-1}}-sqrt{b_{n-1}})^{2times 2}=ldots=2^{-1-2-ldots-n}(sqrt{x}-sqrt{y})^{2n}=2^{-n(n+1)/2}(sqrt{x}-sqrt{y})^{2n}rightarrow 0$ as $nrightarrowinfty.$






share|cite|improve this answer























  • How did you get the third equality?
    – mathnoob
    Nov 23 '18 at 14:09














0












0








0






$|a_{n+1}-b_{n+1}|=left|frac{a_n+b_n}{2}-sqrt{a_nb_n}right|=1/2left|a_n+b_n-2sqrt{a_nb_n}right|=1/2(sqrt{a_n}-sqrt{b_n})^2=(1/2)(1/4)(sqrt{a_{n-1}}-sqrt{b_{n-1}})^4=2^{-1-2}(sqrt{a_{n-1}}-sqrt{b_{n-1}})^{2times 2}=ldots=2^{-1-2-ldots-n}(sqrt{x}-sqrt{y})^{2n}=2^{-n(n+1)/2}(sqrt{x}-sqrt{y})^{2n}rightarrow 0$ as $nrightarrowinfty.$






share|cite|improve this answer














$|a_{n+1}-b_{n+1}|=left|frac{a_n+b_n}{2}-sqrt{a_nb_n}right|=1/2left|a_n+b_n-2sqrt{a_nb_n}right|=1/2(sqrt{a_n}-sqrt{b_n})^2=(1/2)(1/4)(sqrt{a_{n-1}}-sqrt{b_{n-1}})^4=2^{-1-2}(sqrt{a_{n-1}}-sqrt{b_{n-1}})^{2times 2}=ldots=2^{-1-2-ldots-n}(sqrt{x}-sqrt{y})^{2n}=2^{-n(n+1)/2}(sqrt{x}-sqrt{y})^{2n}rightarrow 0$ as $nrightarrowinfty.$







share|cite|improve this answer














share|cite|improve this answer



share|cite|improve this answer








edited Nov 23 '18 at 14:13

























answered Nov 23 '18 at 14:00









John_WickJohn_Wick

1,486111




1,486111












  • How did you get the third equality?
    – mathnoob
    Nov 23 '18 at 14:09


















  • How did you get the third equality?
    – mathnoob
    Nov 23 '18 at 14:09
















How did you get the third equality?
– mathnoob
Nov 23 '18 at 14:09




How did you get the third equality?
– mathnoob
Nov 23 '18 at 14:09


















draft saved

draft discarded




















































Thanks for contributing an answer to Mathematics Stack Exchange!


  • Please be sure to answer the question. Provide details and share your research!

But avoid



  • Asking for help, clarification, or responding to other answers.

  • Making statements based on opinion; back them up with references or personal experience.


Use MathJax to format equations. MathJax reference.


To learn more, see our tips on writing great answers.




draft saved


draft discarded














StackExchange.ready(
function () {
StackExchange.openid.initPostLogin('.new-post-login', 'https%3a%2f%2fmath.stackexchange.com%2fquestions%2f3010377%2fprove-that-two-sequences-converge-to-the-same-limit%23new-answer', 'question_page');
}
);

Post as a guest















Required, but never shown





















































Required, but never shown














Required, but never shown












Required, but never shown







Required, but never shown

































Required, but never shown














Required, but never shown












Required, but never shown







Required, but never shown







Popular posts from this blog

Biblatex bibliography style without URLs when DOI exists (in Overleaf with Zotero bibliography)

How to change which sound is reproduced for terminal bell?

Title Spacing in Bjornstrup Chapter, Removing Chapter Number From Contents